Variation of Parameters problem

Click For Summary
The discussion revolves around solving the differential equation t²y'' - 2y = 3t² - 1 using the method of variation of parameters. The user obtained a particular solution of Y(t) = t²ln(t) - (1/3)t² + (1/2), while the textbook solution is Y(t) = t²ln(t) + (1/2). There is confusion regarding a discrepancy of a term in the solutions, with the user insisting their integration is correct after multiple attempts. Other participants suggest that the user may have made an error in their second integral, but without detailed work shown, it's difficult to pinpoint the mistake. The user expresses frustration over the time required to present their work clearly, indicating a persistent issue with the problem.
pergradus
Messages
137
Reaction score
1

Homework Statement



Find a particular solution by method of variation of parameters:

t2y'' - 2y = 3t2 - 1

given:

y1 = t2
y2 = t-1

Homework Equations



img9.gif


The Attempt at a Solution



I get Y(t) = t^2ln(t) - \frac{1}{3}t^2 + \frac{1}{2}

The book gives Y(t) = t^2ln(t) + \frac{1}{2}

I don't understand why they are off by the 1 term?
 
Physics news on Phys.org
The only thing I can tell you is that they aren't off by one term- you are!

And since you don't show any work, we can't say why you are. I suspect you have an error in your second integral.
 
HallsofIvy said:
The only thing I can tell you is that they aren't off by one term- you are!

And since you don't show any work, we can't say why you are. I suspect you have an error in your second integral.

By "they" I was referring to the the two answers - mine and theirs.

And I made no mistakes in the integration - check my work if you want. This is the THIRD time I've done this problem and I get the same answer.

It will take me hours to type this in LaTex so please just look at picture I uploaded. The -1/3t2 term just doesn't go away.
 

Attachments

  • CIMG0015.jpg
    CIMG0015.jpg
    16.4 KB · Views: 556
Question: A clock's minute hand has length 4 and its hour hand has length 3. What is the distance between the tips at the moment when it is increasing most rapidly?(Putnam Exam Question) Answer: Making assumption that both the hands moves at constant angular velocities, the answer is ## \sqrt{7} .## But don't you think this assumption is somewhat doubtful and wrong?

Similar threads

  • · Replies 7 ·
Replies
7
Views
2K
  • · Replies 3 ·
Replies
3
Views
2K
  • · Replies 2 ·
Replies
2
Views
915
  • · Replies 2 ·
Replies
2
Views
1K
  • · Replies 2 ·
Replies
2
Views
1K
  • · Replies 1 ·
Replies
1
Views
2K
  • · Replies 9 ·
Replies
9
Views
2K
  • · Replies 13 ·
Replies
13
Views
1K
  • · Replies 3 ·
Replies
3
Views
2K
  • · Replies 11 ·
Replies
11
Views
2K